summaryrefslogtreecommitdiff
path: root/exercise02
diff options
context:
space:
mode:
authorPhilipp Le <philipp-le-prviat@freenet.de>2020-05-18 23:33:06 +0200
committerPhilipp Le <philipp-le-prviat@freenet.de>2021-03-04 01:16:19 +0100
commit0ab48e46d376c406647a161bb184e4e671a930e7 (patch)
tree514d9e492b1beb9c4f147a70699d3f8804d2967f /exercise02
parent8bfa77653058d2409d87bd2b836ef3ac33417a97 (diff)
downloaddcs-lecture-notes-0ab48e46d376c406647a161bb184e4e671a930e7.zip
dcs-lecture-notes-0ab48e46d376c406647a161bb184e4e671a930e7.tar.gz
dcs-lecture-notes-0ab48e46d376c406647a161bb184e4e671a930e7.tar.bz2
Exercise 2 rev 1
Diffstat (limited to 'exercise02')
-rw-r--r--exercise02/exercise02.tex69
1 files changed, 63 insertions, 6 deletions
diff --git a/exercise02/exercise02.tex b/exercise02/exercise02.tex
index 51099f7..580ac67 100644
--- a/exercise02/exercise02.tex
+++ b/exercise02/exercise02.tex
@@ -37,7 +37,7 @@
\end{tasks}
\end{solution}
-\begin{question}
+\begin{question}[subtitle={Fourier Series}]
The following periodic signal is given.
\begin{figure}[H]
\centering
@@ -315,7 +315,7 @@
\end{tasks}
\end{solution}
-\begin{question}
+\begin{question}[subtitle={System Analysis}]
The following circuit is given.
\begin{figure}[H]
\centering
@@ -333,8 +333,8 @@
Find a differential equation which connects $u_i(t)$ and $u_o(t)$!
\task
Determine the transfer function $\underline{H} \left(j \omega\right)$!
- \task
- Calculate the impulse response!
+ %\task
+ %Calculate the impulse response!
\task
Is the system causal? Why?
\task
@@ -342,7 +342,11 @@
\end{tasks}
\end{question}
-\begin{question}
+\begin{solution}
+ %TODO
+\end{solution}
+
+\begin{question}[subtitle={Amplitude and Phase Response}]
\begin{figure}[H]
\centering
\begin{circuitikz}
@@ -377,7 +381,7 @@
\task
The following signal is applied to the input of the system $u_i(t)$.
\begin{equation}
- u_i(t) = \SI{2}{V} \cos\left(2 \pi \cdot \SI{25}{kHz} \cdot t\right)
+ u_i(t) = \SI{2}{V} \cdot \cos\left(2 \pi \cdot \SI{2.5}{kHz} \cdot t\right)
\end{equation}
Calculate the output signal $u_o(t)$ as either a time domain function or a phasor!
\end{tasks}
@@ -407,6 +411,59 @@
\end{itemize}
The system is stable because the real part of its pole is negative.
+ \task
+ \begin{equation*}
+ \begin{split}
+ \underline{H}\left(j \omega\right) &= \frac{j \omega RC}{j \omega RC + 1} \\
+ &= \frac{j \omega RC \left(j \omega RC - 1\right)}{\left(j \omega RC + 1\right)\left(j \omega RC - 1\right)} \\
+ &= \frac{-\left(\omega RC\right)^2 - j \omega RC}{- \left(j \omega RC\right)^2 - 1} \\
+ &= \frac{\left(\omega RC\right)^2 + j \omega RC}{\left(j \omega RC\right)^2 + 1} \\
+ \Re\left\{\underline{H}\left(j \omega\right)\right\} &= \frac{\left(\omega RC\right)^2}{\left(j \omega RC\right)^2 + 1} \\
+ \Im\left\{\underline{H}\left(j \omega\right)\right\} &= \frac{\omega RC}{\left(j \omega RC\right)^2 + 1}
+ \end{split}
+ \end{equation*}
+ \begin{equation*}
+ \begin{split}
+ \left|\underline{H}\left(j \omega\right)\right| &= \sqrt{\frac{\left(\omega RC\right)^4 + \left(\omega RC\right)^2}{\left(\left(j \omega RC\right)^2 + 1\right)^2}} \\
+ &= \sqrt{\frac{\left(\omega RC\right)^2 \left(\left(j \omega RC\right)^2 + 1\right)}{\left(\left(j \omega RC\right)^2 + 1\right)^2}} \\
+ &= \sqrt{\frac{\left(\omega RC\right)^2}{\left(j \omega RC\right)^2 + 1}}
+ \end{split}
+ \end{equation*}
+
+ \task
+ \begin{equation*}
+ \begin{split}
+ \arg\left(\underline{H}\left(j \omega\right)\right) &= \mathrm{atan2}\left(\Im\left\{\underline{H}\left(j \omega\right)\right\}, \Re\left\{\underline{H}\left(j \omega\right)\right\}\right) \\
+ &= \begin{cases}
+ \arctan\left(\frac{\Im\left\{\underline{H}\left(j \omega\right)\right\}}{\Re\left\{\underline{H}\left(j \omega\right)\right\}}\right) &\quad \text{ if } \Re\left\{\underline{H}\left(j \omega\right)\right\} > 0, \\
+ \arctan\left(\frac{\Im\left\{\underline{H}\left(j \omega\right)\right\}}{\Re\left\{\underline{H}\left(j \omega\right)\right\}}\right) + \pi &\quad \text{ if } \Re\left\{\underline{H}\left(j \omega\right)\right\} < 0 \text{ and } \Im\left\{\underline{H}\left(j \omega\right)\right\} \geq 0, \\
+ \arctan\left(\frac{\Im\left\{\underline{H}\left(j \omega\right)\right\}}{\Re\left\{\underline{H}\left(j \omega\right)\right\}}\right) - \pi &\quad \text{ if } \Re\left\{\underline{H}\left(j \omega\right)\right\} < 0 \text{ and } \Im\left\{\underline{H}\left(j \omega\right)\right\} < 0, \\
+ +\frac{\pi}{2} &\quad \text{ if } \Re\left\{\underline{H}\left(j \omega\right)\right\} = 0 \text{ and } \Im\left\{\underline{H}\left(j \omega\right)\right\} > 0, \\
+ -\frac{\pi}{2} &\quad \text{ if } \Re\left\{\underline{H}\left(j \omega\right)\right\} = 0 \text{ and } \Im\left\{\underline{H}\left(j \omega\right)\right\} < 0, \\
+ \text{undefined} &\quad \text{ if } \Re\left\{\underline{H}\left(j \omega\right)\right\} = 0 \text{ and } \Im\left\{\underline{H}\left(j \omega\right)\right\} = 0. \\
+ \end{cases} \\
+ &= \arctan\left(\frac{\Im\left\{\underline{H}\left(j \omega\right)\right\}}{\Re\left\{\underline{H}\left(j \omega\right)\right\}}\right)
+ \end{split}
+ \end{equation*}
+
+ \task
+ \begin{equation*}
+ \underline{U}_i\left(j \omega\right) = \mathcal{F}\left\{u_i(t)\right\} = \SI{2}{V} \pi \left(\delta\left(\omega - 2 \pi \cdot \SI{2.5}{kHz}\right) + \delta\left(\omega + 2 \pi \cdot \SI{2.5}{kHz}\right)\right)
+ \end{equation*}
+ \begin{equation*}
+ \begin{split}
+ \underline{U}_i\left(j \omega\right) &= \underline{H}\left(j \omega\right) \underline{U}_i\left(j \omega\right) \\
+ &= \SI{2}{V} \pi \left(\underline{H}\left(j \left(2 \pi \cdot \SI{2.5}{kHz}\right)\right) \delta\left(\omega - 2 \pi \cdot \SI{2.5}{kHz}\right) \right. + \\ &\qquad \left. \underline{H}\left(-j \left(2 \pi \cdot \SI{2.5}{kHz}\right)\right) \delta\left(\omega + 2 \pi \cdot \SI{2.5}{kHz}\right)\right) \\
+ &= \SI{2}{V} \pi \left(0.594 \cdot e^{j \SI{53.6}{\degree}} \cdot \delta\left(\omega - 2 \pi \cdot \SI{2.5}{kHz}\right) \right. + \\ &\qquad \left. 0.594 \cdot e^{-j \SI{53.6}{\degree}} \cdot \delta\left(\omega + 2 \pi \cdot \SI{2.5}{kHz}\right)\right) \\
+ &= \SI{1.19}{V} \pi \left(e^{j \SI{53.6}{\degree}} \cdot \delta\left(\omega - 2 \pi \cdot \SI{2.5}{kHz}\right) + e^{-j \SI{53.6}{\degree}} \cdot \delta\left(\omega + 2 \pi \cdot \SI{2.5}{kHz}\right)\right)
+ \end{split}
+ \end{equation*}
+ Using the time-shift theorem:
+ \begin{equation*}
+ u_o(t) = \mathcal{F}^{-1}\left\{\underline{U}_i\left(j \omega\right)\right\} = \SI{1.19}{V} \cdot \cos\left(2 \pi \cdot \SI{2.5}{kHz} \cdot t - \SI{53.6}{\degree}\right)
+ \end{equation*}
+ The signal has been attenuated and phase-shifted.
+
%TODO
\end{tasks}
\end{solution}